- PowerScore Staff
- Posts: 5972
- Joined: Mar 25, 2011
- Thu Jun 09, 2016 6:23 pm
#26376
Complete Question Explanation
(The complete setup for this game can be found here: lsat/viewtopic.php?t=6413)
The correct answer choice is (B)
Answer choice (A) is incorrect because it violates the third rule.
Answer choice (B) is the correct answer choice.
Answer choice (C) is incorrect because it violates the third rule.
Answer choice (D) is incorrect because it violates the second rule.
Answer choice (E) is incorrect because it violates the first rule.
(The complete setup for this game can be found here: lsat/viewtopic.php?t=6413)
The correct answer choice is (B)
Answer choice (A) is incorrect because it violates the third rule.
Answer choice (B) is the correct answer choice.
Answer choice (C) is incorrect because it violates the third rule.
Answer choice (D) is incorrect because it violates the second rule.
Answer choice (E) is incorrect because it violates the first rule.
Dave Killoran
PowerScore Test Preparation
Follow me on X/Twitter at http://twitter.com/DaveKilloran
My LSAT Articles: http://blog.powerscore.com/lsat/author/dave-killoran
PowerScore Podcast: http://www.powerscore.com/lsat/podcast/
PowerScore Test Preparation
Follow me on X/Twitter at http://twitter.com/DaveKilloran
My LSAT Articles: http://blog.powerscore.com/lsat/author/dave-killoran
PowerScore Podcast: http://www.powerscore.com/lsat/podcast/